Actual Exam
NR565 Advanced Pharmacology Midterm
Actual Exam Prep Latest 2025/2026 Complete
380 Questions And Correct Detailed Answers
(Verified Answers)
A 39-year-old man who suffered a work-related injury has chronic back pain. A morphine pain pump
was implanted to control his chronic pain. Formerly, he had chronic diarrhea which is now much
improved. What is the most likely explanation for this finding?
(A) Improved gastrointestinal motility
(B) Improved intestinal circular muscle tone
(C) Weakened anal sphincter tone
(D) Weakened rectal balloon dilation pressure
(E) Weakened transverse colonic musculature
- Correct Answer :(B) Improved intestinal circular muscle tone
A 6-year-old boy presents to the emergency department
with an altered mental status. He is hyperventilating, has a rash on his hands, and a high fever. His
mother has been giving him an antipyretic for his fever for the past 2 days. The patient's liver
enzymes are elevated. His mental status continues to decline. What is the mechanism of action of
the most likely medication given to this child by his mother?
(A) Inhibits phospholipase A2
(B) Irreversibly inhibits cyclooxygenases 1 and 2
(C) Reversibly inhibits cyclooxygenases 1 and 2
A+ TEST BANK 1
, NR565 Advanced Pharmacology Midterm
Actual Exam
(D) Reversibly inhibits cyclooxygenase 2
(E) Reversibly inhibits H1 histamine receptors
- Correct Answer :(B) Irreversibly inhibits cyclooxygenases 1 and 2
A 51-year-old alcoholic man presents to the emergency department with severe pain in his right big
toe. The pain is so unbearable that even the sheets touching it at night caused him excruciating pain.
The toe is erythematous, edematous, and tender. The diagnosis of gout is made after negatively
birefringent crystals are seen from joint aspirate. The patient is given colchicine. What is the
mechanism of action of colchicine?
(A) Inhibits microtubule polymerization
(B) Inhibits reabsorption of uric acid
(C) Inhibits xanthine oxidase
(D) Irreversibly inhibits cyclooxygenase
(E) Reversibly inhibits cyclooxygenase
- Correct Answer :(A) Inhibits microtubule polymerization
A 34-year-old man with exercised-induced asthma is searching for a bronchodilator that will allow
him to
run in a marathon. The event will take him approximately 6 h and 30 min to complete. The following
diagram shows five bronchodilators. Which of the following agents will provide him with the most
efficacious therapy during his run?
(A) Letter A
(B) Letter B
(C) Letter C
(D) Letter D
A+ TEST BANK 2
, NR565 Advanced Pharmacology Midterm
Actual Exam
(E) Letter E
- Correct Answer :(D) Letter D
A 38-year-old man who is obese complains of an extremely
painful, swollen metatarsophalangeal joint of his left big toe. He has had two similar attacks in the
past 4 years. The physician prescribes febuxostat.
Which of the following describes febuxostat's mechanism of action?
(A) Anti-inflammatory
(B) Inhibition of leukocyte migration
(C) Inhibition of urate reabsorption
(D) Inhibition of xanthine oxidase
(E) Upregulation of urate metabolism
- Correct Answer :(D) Inhibition of xanthine oxidase
A 52-year-old man with a long history of gastroesophageal reflux disease and peptic ulcer disease has
been
taking multiple medications to control symptoms. His physician places him on misoprostol. This
agent works at which of the following sites?
(A) Letter A
(B) Letter B
(C) Letter C
(D) Letter D
- Correct Answer :(C) Letter C
A+ TEST BANK 3
, NR565 Advanced Pharmacology Midterm
Actual Exam
A 31-year-old woman with a sore throat, cough, and rhinitis presents to the ambulatory care clinic for
evaluation. She has no known allergies, although she has never taken antibiotics. She is given a
prescription for penicillin and develops a maculopapular rash on
her hands and swelling of her tongue. The most likely explanation for these findings is
(A) Arachidonic acid
(B) Macrophages
(C) Penicillin
(D) Penicilloic acid
(E) Xanthine oxidase
- Correct Answer :(D) Penicilloic acid
A 43-year-old man with overwhelming sepsis has been receiving intravenous gentamicin for 3
months. He now complains of hearing loss. What is the most likely explanation for this finding?
(A) Destroyed hair cells in the organ of Corti
(B) Edema of the pinnae
(C) Low peak plasma levels but toxic to the ear
(D) Otitis externa
(E) Vestibular toxicity
- Correct Answer :(A) Destroyed hair cells in the organ of Corti
A 28-year-old woman presents to the emergency department
in an acute asthma exacerbation. Her
A+ TEST BANK 4